Math, asked by kasaadarsh, 6 months ago


38. The ratio in which the line segment joining A(6.3) and B(-2,-5) is divided by the
X-axis is
a. 2:3
b. 3:2
c. 3:5
d. 1:3​

Answers

Answered by Anonymous
2

Given ,

The line segment joining A(6,3) and B(-2,-5) is divided by the x axis

Let , the ratio in which x axis divides the line segment AB is m : n

We know that , the section formula is given by

 \boxed{ \tt{x =  \frac{m x_{2} + n x_{1} }{m + n}  \:  \:,  \:  \: y=  \frac{m y_{2} + n y_{1} }{m + n}  }}

And the coordinate of x axis = (x , 0)

Thus ,

 \tt \implies 0 =  \frac{ - 5m + 3n}{m + n}

\tt \implies 5m = 3n

 \tt \implies\frac{m}{n}  =  \frac{3}{5}

Therefore , the correct option is C

_______________ Keep Smiling ☺

Answered by sandhyamalladi121
1

Given ,

The line segment joining A(6,3) and B(-2,-5) is divided by the x axis

Let , the ratio in which x axis divides the line segment AB is m : n

We know that , the section formula is given by

\boxed{ \tt{x = \frac{m x_{2} + n x_{1} }{m + n} \: \:, \: \: y= \frac{m y_{2} + n y_{1} }{m + n} }} </p><p></p><p> </p><p>	</p><p> </p><p>

And the coordinate of x axis = (x , 0)

Thus ,

\tt \implies 0 = \frac{ - 5m + 3n}{m + n}</p><p>  \\ </p><p></p><p>\tt \implies 5m = 3n \\ </p><p></p><p>\tt \implies\frac{m}{n} = \frac{3}{5}

Therefore , the correct option is C

_______________ Keep Smiling ☺

Similar questions